Finance Chapter 2 City Tba Topics Risk Aversion Keywords Blooms Analysis Other Type Multiple Choice

subject Type Homework Help
subject Pages 14
subject Words 10251
subject Authors Eugene F. Brigham, Phillip R. Daves

Unlock document.

This document is partially blurred.
Unlock all pages and 1 million more documents.
Get Access
page-pf1
Chapter 02: Risk and Return: Part I
KEYWORDS:
Bloom’s: Comprehension
49. The CAPM is a multi-period model that takes account of differences in securities' maturities, and it can be used to
determine the required rate of return for any given level of systematic risk.
a.
True
b.
False
ANSWER:
False
50. If markets are in equilibrium, which of the following conditions will exist?
a.
b.
c.
d.
e.
ANSWER:
a
page-pf2
51. You are considering investing in one of the these three stocks:
Stock
Standard Deviation
Beta
A
20%
0.59
B
10%
0.61
C
12%
1.29
If you are a strict risk minimizer, you would choose Stock ____ if it is to be held in isolation and Stock ____ if it is to be
held as part of a well-diversified portfolio.
a.
A; B.
b.
B; A.
c.
C; A.
d.
C; B.
e.
A; A.
ANSWER:
b
52. Your friend is considering adding one additional stock to a 3-stock portfolio, to form a 4-stock portfolio. She is highly
risk averse and has asked for your advice. The three stocks currently held all have b = 1.0, and they are perfectly
positively correlated with the market. Potential new Stocks A and B both have expected returns of 15%, are in
equilibrium, and are equally correlated with the market, with r = 0.75. However, Stock A's standard deviation of returns is
12% versus 8% for Stock B. Which stock should this investor add to his or her portfolio, or does the choice not matter?
a.
Stock A.
b.
Stock B.
c.
Neither A nor B, as neither has a return sufficient to compensate for risk.
d.
Add A, since its beta must be lower.
e.
Either A or B, i.e., the investor should be indifferent between the two.
ANSWER:
b
page-pf3
53. Which of the following is NOT a potential problem when estimating and using betas, i.e., which statement is FALSE?
a.
Sometimes, during a period when the company is undergoing a change such as toward more leverage or riskier
assets, the calculated beta will be drastically different from the "true" or "expected future" beta.
b.
The beta of an "average stock," or "the market," can change over time, sometimes drastically.
c.
Sometimes the past data used to calculate beta do not reflect the likely risk of the firm for the future because
conditions have changed.
d.
All of the statements above are true.
e.
The fact that a security or project may not have a past history that can be used as the basis for calculating beta.
ANSWER:
b
54. Stock A's beta is 1.7 and Stock B's beta is 0.7. Which of the following statements must be true about these securities?
(Assume market equilibrium.)
a.
Stock B must be a more desirable addition to a portfolio than A.
b.
Stock A must be a more desirable addition to a portfolio than B.
c.
The expected return on Stock A should be greater than that on B.
d.
The expected return on Stock B should be greater than that on A.
e.
When held in isolation, Stock A has more risk than Stock B.
ANSWER:
c
page-pf4
Copyright Cengage Learning. Powered by Cognero.
Page 24
55. Which of the following statements is CORRECT?
a.
If you found a stock with a zero historical beta and held it as the only stock in your portfolio, you would by
definition have a riskless portfolio.
b.
The beta coefficient of a stock is normally found by regressing past returns on a stock against past market
returns. One could also construct a scatter diagram of returns on the stock versus those on the market, estimate
the slope of the line of best fit, and use it as beta. However, this historical beta may differ from the beta that
exists in the future.
c.
The beta of a portfolio of stocks is always larger than the betas of any of the individual stocks.
d.
It is theoretically possible for a stock to have a beta of 1.0. If a stock did have a beta of 1.0, then, at least in
theory, its required rate of return would be equal to the risk-free (default-free) rate of return, rRF.
e.
The beta of a portfolio of stocks is always smaller than the betas of any of the individual stocks.
ANSWER:
b
56. Which of the following statements is CORRECT?
a.
Suppose the returns on two stocks are negatively correlated. One has a beta of 1.2 as determined in a
regression analysis using data for the last 5 years, while the other has a beta of 0.6. The returns on the stock
with the negative beta must have been negatively correlated with returns on most other stocks during that 5-
year period.
b.
Suppose you are managing a stock portfolio, and you have information that leads you to believe the stock
page-pf5
Chapter 02: Risk and Return: Part I
market is likely to be very strong in the immediate future. That is, you are convinced that the market is about
to rise sharply. You should sell your high-beta stocks and buy low-beta stocks in order to take advantage of the
expected market move.
c.
You think that investor sentiment is about to change, and investors are about to become more risk averse. This
suggests that you should re-balance your portfolio to include more high-beta stocks.
d.
If the market risk premium remains constant, but the risk-free rate declines, then the required returns on low-
beta stocks will rise while those on high-beta stocks will decline.
e.
Paid-in-Full Inc. is in the business of collecting past-due accounts for other companies, i.e., it is a collection
agency. Paid-in-Full's revenues, profits, and stock price tend to rise during recessions. This suggests that Paid-
in-Full Inc.'s beta should be quite high, say 2.0, because it does so much better than most other companies
when the economy is weak.
ANSWER:
a
57. Which of the following statements is CORRECT?
a.
Logically, it is easier to estimate the betas associated with capital budgeting projects than the betas associated
with stocks, especially if the projects are closely associated with research and development activities.
b.
The beta of an "average stock," which is also "the market beta," can change over time, sometimes drastically.
c.
If a newly issued stock does not have a past history that can be used for calculating beta, then we should
always estimate that its beta will turn out to be 1.0. This is especially true if the company finances with more
debt than the average firm.
d.
During a period when a company is undergoing a change such as increasing its use of leverage or taking on
riskier projects, the calculated historical beta may be drastically different from the beta that will exist in the
future.
e.
If a company with a high beta merges with a low-beta company, the best estimate of the new merged
company's beta is 1.0.
ANSWER:
d
page-pf6
58. Stock A's beta is 1.7 and Stock B's beta is 0.7. Which of the following statements must be true, assuming the CAPM is
correct.
a.
In equilibrium, the expected return on Stock B will be greater than that on Stock A.
b.
When held in isolation, Stock A has more risk than Stock B.
c.
Stock B would be a more desirable addition to a portfolio than A.
d.
In equilibrium, the expected return on Stock A will be greater than that on B.
e.
Stock A would be a more desirable addition to a portfolio then Stock B.
ANSWER:
d
59. Stock X has a beta of 0.7 and Stock Y has a beta of 1.7. Which of the following statements must be true, according to
the CAPM?
a.
Stock Y's realized return during the coming year will be higher than Stock X's return.
b.
If the expected rate of inflation increases but the market risk premium is unchanged, the required returns on
the two stocks should increase by the same amount.
c.
Stock Y's return has a higher standard deviation than Stock X.
d.
If the market risk premium declines, but the risk-free rate is unchanged, Stock X will have a larger decline in
its required return than will Stock Y.
e.
If you invest $50,000 in Stock X and $50,000 in Stock Y, your 2-stock portfolio would have a beta
significantly lower than 1.0, provided the returns on the two stocks are not perfectly correlated.
ANSWER:
b
page-pf7
Copyright Cengage Learning. Powered by Cognero.
Page 27
60. Consider the following average annual returns for Stocks A and B and the Market. Which of the possible answers best
describes the historical betas for A and B?
Years
Market
Stock A
Stock B
1
0.03
0.16
0.05
2
0.05
0.20
0.05
3
0.01
0.18
0.05
4
0.10
0.25
0.05
5
0.06
0.14
0.05
a.
bA > +1; bB = 0.
b.
bA = 0; bB = 1.
c.
bA < 0; bB = 0.
d.
bA < 1; bB = 1.
e.
bA > 0; bB = 1.
ANSWER:
c
61.
Which of the following statements is CORRECT?
a.
The higher the correlation between the stocks in a portfolio, the lower the risk inherent in the portfolio.
page-pf8
Chapter 02: Risk and Return: Part I
b.
An investor can eliminate almost all risk if he or she holds a very large and well diversified portfolio of stocks.
c.
Once a portfolio has about 40 stocks, adding additional stocks will not reduce its risk by even a small amount.
d.
An investor can eliminate almost all diversifiable risk if he or she holds a very large, well-diversified portfolio
of stocks.
e.
An investor can eliminate almost all market risk if he or she holds a very large and well diversified portfolio of
stocks.
ANSWER:
d
62. Which of the following statements is CORRECT?
a.
If you were restricted to investing in publicly traded common stocks, yet you wanted to minimize the riskiness
of your portfolio as measured by its beta, then according to the CAPM theory you should invest an equal
amount of money in each stock in the market. That is, if there were 10,000 traded stocks in the world, the least
risky possible portfolio would include some shares of each one.
b.
If you formed a portfolio that consisted of all stocks with betas less than 1.0, which is about half of all stocks,
the portfolio would itself have a beta coefficient that is equal to the weighted average beta of the stocks in the
portfolio, and that portfolio would have less risk than a portfolio that consisted of all stocks in the market.
c.
Market risk can be eliminated by forming a large portfolio, and if some Treasury bonds are held in the
portfolio, the portfolio can be made to be completely riskless.
d.
A portfolio that consists of all stocks in the market would have a required return that is equal to the riskless
rate.
e.
If you add enough randomly selected stocks to a portfolio, you can completely eliminate all of the market risk
from the portfolio.
ANSWER:
b
page-pf9
63. Recession, inflation, and high interest rates are economic events that are best characterized as being
a.
company-specific risk factors that can be diversified away.
b.
among the factors that are responsible for market risk.
c.
risks that are beyond the control of investors and thus should not be considered by security analysts or
portfolio managers.
d.
irrelevant except to governmental authorities like the Federal Reserve.
e.
systematic risk factors that can be diversified away.
ANSWER:
b
64. Which of the following statements is CORRECT?
a.
If an investor buys enough stocks, he or she can, through diversification, eliminate all of the diversifiable risk
inherent in owning stocks. Therefore, if a portfolio contained all publicly traded stocks, it would be essentially
riskless.
b.
The required return on a firm's common stock is, in theory, determined solely by its market risk. If the market
risk is known, and if that risk is expected to remain constant, then no other information is required to specify
the firm's required return.
c.
Portfolio diversification reduces the variability of returns (as measured by the standard deviation) of each
individual stock held in a portfolio.
d.
A security's beta measures its non-diversifiable, or market, risk relative to that of an average stock.
e.
A stock's beta is less relevant as a measure of risk to an investor with a well-diversified portfolio than to an
investor who holds only that one stock.
ANSWER:
d
page-pfa
Copyright Cengage Learning. Powered by Cognero.
Page 30
65. Which of the following statements is CORRECT?
a.
Diversifiable risk can be reduced by forming a large portfolio, but normally even highly-diversified portfolios
are subject to market (or systematic) risk.
b.
A large portfolio of randomly selected stocks will have a standard deviation of returns that is greater than the
standard deviation of a 1-stock portfolio if that one stock has a beta less than 1.0.
c.
A large portfolio of stocks whose betas are greater than 1.0 will have less market risk than a single stock with
a beta = 0.8.
d.
If you add enough randomly selected stocks to a portfolio, you can completely eliminate all of the market risk
from the portfolio.
e.
A large portfolio of randomly selected stocks will always have a standard deviation of returns that is less than
the standard deviation of a portfolio with fewer stocks, regardless of how the stocks in the smaller portfolio are
selected.
ANSWER:
a
66. Which of the following statements is CORRECT?
a.
A portfolio that consists of 40 stocks that are not highly correlated with "the market" will probably be less
risky than a portfolio of 40 stocks that are highly correlated with the market, assuming the stocks all have the
same standard deviations.
b.
A two-stock portfolio will always have a lower beta than a one-stock portfolio.
c.
If portfolios are formed by randomly selecting stocks, a 10-stock portfolio will always have a lower beta than
a one-stock portfolio.
d.
A stock with an above-average standard deviation must also have an above-average beta.
e.
A two-stock portfolio will always have a lower standard deviation than a one-stock portfolio.
page-pfb
Chapter 02: Risk and Return: Part I
ANSWER:
a
67. Consider the following information for three stocks, A, B, and C. The stocks' returns are positively but not perfectly
positively correlated with one another, i.e., the correlations are all between 0 and 1.
Expected
Standard
Stock
Return
Deviation
Beta
A
10%
20%
1.0
B
10%
10%
1.0
C
12%
12%
1.4
Portfolio AB has half of its funds invested in Stock A and half in Stock B. Portfolio ABC has one third of its funds
invested in each of the three stocks. The risk-free rate is 5%, and the market is in equilibrium, so required returns equal
expected returns. Which of the following statements is CORRECT?
a.
Portfolio AB's coefficient of variation is greater than 2.0.
b.
Portfolio AB's required return is greater than the required return on Stock A.
c.
Portfolio ABC's expected return is 10.66667%.
d.
Portfolio ABC has a standard deviation of 20%.
e.
Portfolio AB has a standard deviation of 20%.
ANSWER:
c
page-pfc
68. Which of the following statements is CORRECT?
a.
A portfolio with a large number of randomly selected stocks would have more market risk than a single stock
that has a beta of 0.5, assuming that the stock's beta was correctly calculated and is stable.
b.
If a stock has a negative beta, its expected return must be negative.
c.
A portfolio with a large number of randomly selected stocks would have less market risk than a single stock
that has a beta of 0.5.
d.
According to the CAPM, stocks with higher standard deviations of returns must also have higher expected
returns.
e.
If the returns on two stocks are perfectly positively correlated (i.e., the correlation coefficient is +1.0) and
these stocks have identical standard deviations, an equally weighted portfolio of the two stocks will have a
standard deviation that is less than that of the individual stocks.
ANSWER:
a
69. Which of the following is most likely to be true for a portfolio of 40 randomly selected stocks?
a.
The riskiness of the portfolio is the same as the riskiness of each stock if it was held in isolation.
b.
The beta of the portfolio is less than the average of the betas of the individual stocks.
c.
The beta of the portfolio is equal to the average of the betas of the individual stocks.
d.
The beta of the portfolio is larger than the average of the betas of the individual stocks.
e.
The riskiness of the portfolio is greater than the riskiness of each of the stocks if each was held in isolation.
ANSWER:
c
page-pfd
70. If you randomly select stocks and add them to your portfolio, which of the following statements best describes what
you should expect?
a.
Adding more such stocks will increase the portfolio's expected rate of return.
b.
Adding more such stocks will reduce the portfolio's beta coefficient and thus its systematic risk.
c.
Adding more such stocks will have no effect on the portfolio's risk.
d.
Adding more such stocks will reduce the portfolio's market risk but not its unsystematic risk.
e.
Adding more such stocks will reduce the portfolio's unsystematic, or diversifiable, risk.
ANSWER:
e
71. Charlie and Lucinda each have $50,000 invested in stock portfolios. Charlie's has a beta of 1.2, an expected return of
10.8%, and a standard deviation of 25%. Lucinda's has a beta of 0.8, an expected return of 9.2%, and a standard deviation
that is also 25%. The correlation coefficient, r, between Charlie's and Lucinda's portfolios is zero. If Charlie and Lucinda
marry and combine their portfolios, which of the following best describes their combined $100,000 portfolio?
a.
The combined portfolio's beta will be equal to a simple weighted average of the betas of the two individual
portfolios, 1.0; its expected return will be equal to a simple weighted average of the expected returns of the
two individual portfolios, 10.0%; and its standard deviation will be less than the simple average of the two
portfolios' standard deviations, 25%.
b.
The combined portfolio's expected return will be greater than the simple weighted average of the expected
returns of the two individual portfolios, 10.0%.
c.
The combined portfolio's standard deviation will be greater than the simple average of the two portfolios'
standard deviations, 25%.
d.
The combined portfolio's standard deviation will be equal to a simple average of the two portfolios' standard
deviations, 25%.
e.
The combined portfolio's expected return will be less than the simple weighted average of the expected returns
of the two individual portfolios, 10.0%.
ANSWER:
a
page-pfe
72. The two stocks in your portfolio, X and Y, have independent returns, so the correlation between them, rXY is zero.
Your portfolio consists of $50,000 invested in Stock X and $50,000 invested in Stock Y. Both stocks have an expected
return of 15%, betas of 1.6, and standard deviations of 30%. Which of the following statements best describes the
characteristics of your 2-stock portfolio?
a.
Your portfolio has a standard deviation less than 30%, and its beta is greater than 1.6.
b.
Your portfolio has a beta equal to 1.6, and its expected return is 15%.
c.
Your portfolio has a beta greater than 1.6, and its expected return is greater than 15%.
d.
Your portfolio has a standard deviation greater than 30% and a beta equal to 1.6.
e.
Your portfolio has a standard deviation of 30%, and its expected return is 15%.
ANSWER:
b
73. Which of the following is most likely to occur as you add randomly selected stocks to your portfolio, which currently
consists of 3 average stocks?
a.
The expected return of your portfolio is likely to decline.
b.
The diversifiable risk will remain the same, but the market risk will likely decline.
c.
Both the diversifiable risk and the market risk of your portfolio are likely to decline.
d.
The total risk of your portfolio should decline, and as a result, the expected rate of return on the portfolio
should also decline.
e.
The diversifiable risk of your portfolio will likely decline, but the expected market risk should not change.
ANSWER:
e
page-pff
Copyright Cengage Learning. Powered by Cognero.
Page 35
74. Ann has a portfolio of 20 average stocks, and Tom has a portfolio of 2 average stocks. Assuming the market is in
equilibrium, which of the following statements is CORRECT?
a.
The required return on Ann's portfolio will be lower than that on Tom's portfolio because Ann's portfolio will
have less total risk.
b.
Tom's portfolio will have more diversifiable risk, the same market risk, and thus more total risk than Ann's
portfolio, but the required (and expected) returns will be the same on both portfolios.
c.
If the two portfolios have the same beta, their required returns will be the same, but Ann's portfolio will have
less market risk than Tom's.
d.
The expected return on Jane's portfolio must be lower than the expected return on Dick's portfolio because
Jane is more diversified.
e.
Ann's portfolio will have less diversifiable risk and also less market risk than Tom's portfolio.
ANSWER:
b
75. Stocks A and B are quite similar: Each has an expected return of 12%, a beta of 1.2, and a standard deviation of 25%.
The returns on the two stocks have a correlation of 0.6. Portfolio P has 50% in Stock A and 50% in Stock B. Which of the
following statements is CORRECT?
a.
Portfolio P has a standard deviation that is greater than 25%.
b.
Portfolio P has an expected return that is less than 12%.
page-pf10
Chapter 02: Risk and Return: Part I
Copyright Cengage Learning. Powered by Cognero.
Page 36
c.
Portfolio P has a standard deviation that is less than 25%.
d.
Portfolio P has a beta that is less than 1.2.
e.
Portfolio P has a beta that is greater than 1.2.
ANSWER:
c
76. Stocks A, B, and C are similar in some respects: Each has an expected return of 10% and a standard deviation of 25%.
Stocks A and B have returns that are independent of one another; i.e., their correlation coefficient, r, equals zero. Stocks A
and C have returns that are negatively correlated with one another; i.e., r is less than 0. Portfolio AB is a portfolio with
half of its money invested in Stock A and half in Stock B. Portfolio AC is a portfolio with half of its money invested in
Stock A and half invested in Stock C. Which of the following statements is CORRECT?
a.
Portfolio AC has an expected return that is greater than 25%.
b.
Portfolio AB has a standard deviation that is greater than 25%.
c.
Portfolio AB has a standard deviation that is equal to 25%.
d.
Portfolio AC has a standard deviation that is less than 25%.
e.
Portfolio AC has an expected return that is less than 10%.
ANSWER:
d
77. Stocks A and B each have an expected return of 15%, a standard deviation of 20%, and a beta of 1.2. The returns on
page-pf11
Chapter 02: Risk and Return: Part I
the two stocks have a correlation coefficient of +0.6. Your portfolio consists of 50% A and 50% B. Which of the
following statements is CORRECT?
a.
The portfolio's expected return is 15%.
b.
The portfolio's standard deviation is greater than 20%.
c.
The portfolio's beta is greater than 1.2.
d.
The portfolio's standard deviation is 20%.
e.
The portfolio's beta is less than 1.2.
ANSWER:
a
78. Stock A has a beta of 0.8, Stock B has a beta of 1.0, and Stock C has a beta of 1.2. Portfolio P has 1/3 of its value
invested in each stock. Each stock has a standard deviation of 25%, and their returns are independent of one another, i.e.,
the correlation coefficients between each pair of stocks is zero. Assuming the market is in equilibrium, which of the
following statements is CORRECT?
a.
Portfolio P's expected return is equal to the expected return on Stock A.
b.
Portfolio P's expected return is less than the expected return on Stock B.
c.
Portfolio P's expected return is equal to the expected return on Stock B.
d.
Portfolio P's expected return is greater than the expected return on Stock C.
e.
Portfolio P's expected return is greater than the expected return on Stock B.
ANSWER:
c
page-pf12
79. In a portfolio of three randomly selected stocks, which of the following could NOT be true; i.e., which statement is
false?
a.
The standard deviation of the portfolio is greater than the standard deviation of one or two of the stocks.
b.
The beta of the portfolio is lower than the lowest of the three betas.
c.
The beta of the portfolio is equal to one of the three stock's betas.
d.
The beta of the portfolio is equal to 1.
e.
The standard deviation of the portfolio is less than the standard deviation of each of the stocks if they were
held in isolation.
ANSWER:
b
80. Stock A has a beta = 0.8, while Stock B has a beta = 1.6. Which of the following statements is CORRECT?
a.
If the marginal investor becomes more risk averse, the required return on Stock B will increase by more than
the required return on Stock A.
b.
An equally weighted portfolio of Stocks A and B will have a beta lower than 1.2.
c.
If the marginal investor becomes more risk averse, the required return on Stock A will increase by more than
the required return on Stock B.
d.
If the risk-free rate increases but the market risk premium remains constant, the required return on Stock A
will increase by more than that on Stock B.
e.
Stock B's required return is double that of Stock A's.
ANSWER:
a
page-pf13
81. Stock A has an expected return of 12%, a beta of 1.2, and a standard deviation of 20%. Stock B also has a beta of 1.2,
but its expected return is 10% and its standard deviation is 15%. Portfolio AB has $300,000 invested in Stock A and
$100,000 invested in Stock B. The correlation between the two stocks' returns is zero (that is, rA,B = 0). Which of the
following statements is CORRECT?
a.
The stocks are not in equilibrium based on the CAPM; if A is valued correctly, then B is overvalued.
b.
The stocks are not in equilibrium based on the CAPM; if A is valued correctly, then B is undervalued.
c.
Portfolio AB's expected return is 11.0%.
d.
Portfolio AB's beta is less than 1.2.
e.
Portfolio AB's standard deviation is 17.5%.
ANSWER:
a
82. You have a portfolio P that consists of 50% Stock X and 50% Stock Y. Stock X has a beta of 0.7 and Stock Y has a
beta of 1.3. The standard deviation of each stock's returns is 20%. The stocks' returns are independent of each other, i.e.,
the correlation coefficient, r, between them is zero. Given this information, which of the following statements is
CORRECT?
a.
The required return on Portfolio P is equal to the market risk premium (rM rRF).
b.
Portfolio P has a beta of 0.7.
c.
Portfolio P has a beta of 1.0 and a required return that is equal to the riskless rate, rRF.
d.
Portfolio P has the same required return as the market (rM).
e.
Portfolio P has a standard deviation of 20%.
ANSWER:
d
page-pf14
Copyright Cengage Learning. Powered by Cognero.
Page 40
83. Which of the following statements is CORRECT? (Assume that the risk-free rate is a constant.)
a.
The effect of a change in the market risk premium depends on the slope of the yield curve.
b.
If the market risk premium increases by 1%, then the required return on all stocks will rise by 1%.
c.
If the market risk premium increases by 1%, then the required return will increase by 1% for a stock that has a
beta of 1.0.
d.
The effect of a change in the market risk premium depends on the level of the risk-free rate.
e.
If the market risk premium increases by 1%, then the required return will increase for stocks that have a beta
greater than 1.0, but it will decrease for stocks that have a beta less than 1.0.
ANSWER:
c
84. In historical data, we see that investments with the highest average annual returns also tend to have the highest
standard deviations of annual returns. This observation supports the notion that there is a positive correlation between risk
and return. Which of the following answers correctly ranks investments from highest to lowest risk (and return), where the
security with the highest risk is shown first, the one with the lowest risk last?
a.
Large-company stocks, small-company stocks, long-term corporate bonds, U.S. Treasury bills, long-term
government bonds.
b.
Small-company stocks, large-company stocks, long-term corporate bonds, long-term government bonds, U.S.
Treasury bills.
c.
U.S. Treasury bills, long-term government bonds, long-term corporate bonds, small-company stocks, large-
company stocks.
d.
Large-company stocks, small-company stocks, long-term corporate bonds, long-term government bonds, U.S.
Treasury bills.
e.
Small-company stocks, long-term corporate bonds, large-company stocks, long-term government bonds, U.S.

Trusted by Thousands of
Students

Here are what students say about us.

Copyright ©2022 All rights reserved. | CoursePaper is not sponsored or endorsed by any college or university.